2008 AMC 12B Problems/Problem 21

Revision as of 01:38, 2 March 2008 by Castle (talk | contribs) (New page: ==Problem 21== Two circles of radius 1 are to be constructed as follows. The center of circle <math>A</math> is chosen uniformly and at random from the line segment joining <math>(0,0)</ma...)
(diff) ← Older revision | Latest revision (diff) | Newer revision → (diff)

Problem 21

Two circles of radius 1 are to be constructed as follows. The center of circle $A$ is chosen uniformly and at random from the line segment joining $(0,0)$ and $(2,0)$. The center of circle $B$ is chosen uniformly and at random, and independently of the first choice, from the line segment joining $(0,1)$ to $(2,1)$. What is the probability that circles $A$ and $B$ intersect?

$\textbf{(A)} \; \frac {2 + \sqrt {2}}{4} \qquad \textbf{(B)} \; \frac {3\sqrt {3} + 2}{8} \qquad \textbf{(C)} \; \frac {2 \sqrt {2} - 1}{2} \qquad \textbf{(D)} \; \frac {2 + \sqrt {3}}{4} \qquad \textbf{(E)} \; \frac {4 \sqrt {3} - 3}{4}$

Solution

Two circles intersect if the distance between their centers is less than the sum of their radii. In this problem, $A$ and $B$ intersect iff $\sqrt{(A_X-B_X)^2+(A_Y-B_Y)^2}\leq2 \Rightarrow (A_X-B_X)^2-1\leq4 \Rightarrow (A_X-B_X)\leq \sqrt{3}$

In other words, the two chosen X-coordinates must differ by no more than $\sqrt{3}$. To find this probability, we divide the problem into cases:

1) $A_X$ is on the interval $(0,2-\sqrt{3})$. The probability that $B_X$ falls within the desired range for a given $A_X$ is $A_X$ (on the left) $+\sqrt{3}$ (on the right) all over 2 (the range of possible values). The total probability for this range is the sum of all these probabilities of $B_X$ (over the range of $A_X$) divided by the total range of $A_X (2)$. Thus, the total possibility for this interval is $\frac{1}{2}(\int_{0}^{2-\sqrt{3}} \frac{x+\sqrt{3}}{2}\,dx\}) =\frac{1}{2}(x^2/4+\frac{x\sqrt{3}}{2} \Big |_{x=0}^{x=2-\sqrt{3}}) = \frac{1}{2}(\frac{4-4\sqrt{3}+3}{4} + \sqrt{3}-\frac{3}{2}) = .125$.


2) $A_X$ is on the interval $(2-\sqrt{3},\sqrt{3})$. In this case, any value of $B_X$ will do, so the probability for the interval is simply $\frac{\sqrt{3}-(2-\sqrt{3})}{2} = \sqrt{3}-1$.


3) $A_X$ is on the interval $(\sqrt{3},2)$. This is identical, by symmetry, to case 1.


The total probability is therefore $\sqrt{3}-1+.25 = \frac{4\sqrt{3}-3}{4}$, which is answer choice E.

See Also

2008 AMC 12B (ProblemsAnswer KeyResources)
Preceded by
Problem 20
Followed by
Problem 22
1 2 3 4 5 6 7 8 9 10 11 12 13 14 15 16 17 18 19 20 21 22 23 24 25
All AMC 12 Problems and Solutions